LSAT 8 – Section 4 – Question 11

You need a full course to see this video. Enroll now and get started in less than a minute.

Target time: 1:14

This is question data from the 7Sage LSAT Scorer. You can score your LSATs, track your results, and analyze your performance with pretty charts and vital statistics - all with a Free Account ← sign up in less than 10 seconds

Question
QuickView
Type Tags Answer
Choices
Curve Question
Difficulty
Psg/Game/S
Difficulty
Explanation
PT8 S4 Q11
+LR
Strengthen +Streng
A
65%
165
B
2%
160
C
2%
159
D
1%
164
E
31%
160
138
154
170
+Harder 147.662 +SubsectionMedium
This page shows a recording of a live class. We're working hard to create our standard, concise explanation videos for the questions in this PrepTest. Thank you for your patience!

Here we have a strengthening questions: Which one of the following most strongly supports the explanation given in the argument?

Our stimulus begins with a phenomenon; distemper virus has caused two thirds of the seal population of the north sea to die since 1988. Our author then offers the hypothesis that the reason the normally latent virus has acted up is that pollution weakened the seals immune systems. Since this is a strengthening question and we have a hypothesis, a great answer would be one which eliminates an alternative explanation for why the virus killed so many seals, or introduces more information consistent with the hypothesis. Let’s see what we get!

Correct Answer Choice (A) If pollution is weakening the immune system of the seals, we would expect it might be doing this to other animals! This answer introduces a result we would expect if our hypothesis was true, and therefore strengthens it.

Answer Choice (B) Good for them, but this doesn’t strengthen our hypothesis. If anything it weakens it since our hypothesis requires “severe” pollution and this suggests pollution could be much better than it was before.

Answer Choice (C) I certainly hope the pollution isn’t that bad then, but this does nothing for our argument.

Answer Choice (D) This is just a factoid.

Answer Choice (E) This weakens our argument by introducing an alternate explanation.

Take PrepTest

Review Results

Leave a Reply